Funny Fibonacci

The familiar Fibonacci sequence ( F n ) = ( 1 , 1 , 2 , 3 , 5 , 8 , ) (F_n) = (1, 1, 2, 3, 5, 8, \ldots) is defined recursively as F 0 = 1 , F 1 = 1 , F n = F n 1 + F n 2 for n 2. F_0 = 1,\ F_1 = 1,\ F_n = F_{n-1} + F_{n-2}\ \text{for}\ n \geq 2. Interestingly, F 25 = 121 393 F_{25} = 121\:393 and F 50 = 20 365 011 074 F_{50} = 20\:365\:011\:074 .

Now define a new sequence ( G n ) (G_n) as follows: G 0 = 1 , G 1 = 2 , G n = 3 G n 1 G n 2 for n 2. G_0 = 1,\ G_1 = 2,\ G_n = 3G_{n-1} - G_{n-2}\ \text{for}\ n \geq 2. Determine G 25 G_{25} without using a calculator.


The answer is 20365011074.

This section requires Javascript.
You are seeing this because something didn't load right. We suggest you, (a) try refreshing the page, (b) enabling javascript if it is disabled on your browser and, finally, (c) loading the non-javascript version of this page . We're sorry about the hassle.

2 solutions

Arjen Vreugdenhil
Jan 19, 2017

The trick is to notice that G n = F 2 n G_n = F_{2n} . It follows immediately that G 25 = F 50 = 20 365 011 074 G_{25} = F_{50} = \boxed{20\:365\:011\:074} .

Proof by induction: It is easy to see that G 0 = F 0 = 1 G_0 = F_0 = 1 and G 1 = F 2 = 2 G_1 = F_2 = 2 .

Induction step: Let n 2 n \geq 2 , and suppose that we know G m = F 2 m G_m = F_{2m} for all m < n m < n . Then G n = 3 G n 1 G n 2 = 3 F 2 n 2 F 2 n 4 = 2 F 2 n 2 + ( F 2 n 3 + F 2 n 4 ) F 2 n 4 = 2 F 2 n 2 + F 2 n 3 = F 2 n 2 + F 2 n 1 = F 2 n . G_n = 3G_{n-1} - G_{n-2} \\ = 3F_{2n-2} - F_{2n-4} \\ = 2F_{2n-2} + (F_{2n-3} + F_{2n-4}) - F_{2n-4} \\ = 2F_{2n-2} + F_{2n-3} \\ = F_{2n-2} + F_{2n-1} \\ = F_{2n}.

Challenge : Generalize this. The recursion equation for the sequence ( G ( k ) ) (G^{(k)}) defined by G n ( k ) = F k n G^{(k)}_n = F_{kn} has the form G n ( k ) = α k G n 1 ( k ) + β k G n 2 ( k ) . G^{(k)}_n = \alpha_k G^{(k)}_{n-1} + \beta_k G^{(k)}_{n-2}. How can α k , β k \alpha_k, \beta_k be calculated? (For instance, α 3 = 4 \alpha_3 = 4 and β 3 = 1 \beta_3 = 1 .)

The solution of the challenge appears to be: α k = L k , β k = ( 1 ) k 1 , \alpha_k = L_k,\ \ \ \beta_k = (-1)^{k-1}, where L n L_n is the Lucas sequence, defined by the same recursion as the Fibonacci sequence but with different initial values: L 1 = 1 , L 2 = 3 , L n = L n 1 + L n 2 . L_1 = 1,\ L_2 = 3,\ L_n = L_{n-1} + L_{n-2}. Can anyone provide proof? I.e. show that in general F n k = L k F n k k + ( 1 ) k 1 F n k 2 k . F_{nk} = L_k F_{nk - k} + (-1)^{k-1} F_{nk - 2k}.

Arjen Vreugdenhil - 4 years, 4 months ago

Log in to reply

consider the equation

F n + k + ( 1 ) k F n k = F n L k F_{n+k}+(-1)^{k}F_{n-k}=F_{n}L_{k} ( 1 ) \Rightarrow(1)

let us try proving this by induction

for k=1 it becomes

F n + 1 + ( 1 ) F n 1 = F n L 1 F_{n+1}+(-1)F_{n-1}=F_{n}L_{1} ,since L 1 = 1 L_{1}=1 it is valid

now lets assume its true for k=m,i.e.

F n + m + ( 1 ) m F n m = F n L m F_{n+m}+(-1)^{m}F_{n-m}=F_{n}L_{m}

now we meed to prove its true for n=m+1

F n + ( m + 1 ) + ( 1 ) ( m + 1 ) F n ( m + 1 ) = F n L m + 1 F_{n+(m+1)}+(-1)^{(m+1)}F_{n-(m+1)}=F_{n}L_{m+1}

we have, F n + ( m + 1 ) = F n + m + F n + ( m 1 ) F_{n+(m+1)}=F_{n+m}+F_{n+(m-1)}

and , F n ( m + 1 ) = F n ( m 1 ) F n m F_{n-(m+1)}=F_{n-(m-1)}-F_{n-m}

substituting these values in the LHS of the equation we get,

F n + m + F n + ( m 1 ) + ( 1 ) ( m + 1 ) ( F n ( m 1 ) F n m ) = ( F n + m + ( 1 ) ( m + 2 ) F n m ) + ( F n + ( m 1 ) + ( 1 ) ( m + 1 ) F n ( m 1 ) ) F_{n+m}+F_{n+(m-1)}+(-1)^{(m+1)}(F_{n-(m-1)}-F_{n-m})=(F_{n+m}+(-1)^{(m+2)}F_{n-m})+(F_{n+(m-1)}+(-1)^{(m+1)}F_{n-(m-1)})

this can be slightly modified to obtain,

( F n + m + ( 1 ) ( m ) F n m ) + ( F n + ( m 1 ) + ( 1 ) ( m 1 ) F n ( m 1 ) ) (F_{n+m}+(-1)^{(m)}F_{n-m})+(F_{n+(m-1)}+(-1)^{(m-1)}F_{n-(m-1)})

which by our assumption is equivalent to

F n L m + F n L ( m 1 ) = F n ( L m + L ( m 1 ) ) = F n L ( m + 1 ) F_{n}L_{m}+F_{n}L_{(m-1)}=F_{n}(L_{m}+L_{(m-1)})=F_{n}L_{(m+1)}

thus it is proved by induction.

now in the above equation let n =p(q-1) and k=p ,we get

F p ( q 1 ) + p + ( 1 ) p F p ( q 1 ) p = F p ( q 1 ) L p F_{p(q-1)+p}+(-1)^{p} F_{p(q-1)-p}=F_{p(q-1)}L_{p}

on simplifying we get,

F p q + ( 1 ) p F p ( q 2 ) = F p ( q 1 ) L q F_{pq}+(-1)^{p} F_{p(q-2)}=F_{p(q-1)}L_{q}

or F p q = F p ( q 1 ) L q + ( 1 ) ( p 1 ) F p ( q 2 ) F_{pq}=F_{p(q-1)}L_{q}+(-1)^{(p-1)} F_{p(q-2)}

Anirudh Sreekumar - 4 years, 4 months ago

Log in to reply

Good ideas here.

Another approach would be to prove from the closed form of these numbers.

Calvin Lin Staff - 4 years, 4 months ago

Log in to reply

@Calvin Lin thank you :)

Anirudh Sreekumar - 4 years, 4 months ago
Ashish Gupta
Feb 8, 2017

0 pending reports

×

Problem Loading...

Note Loading...

Set Loading...